Difference between revisions of "Euler's Totient Theorem Problem 1 Solution"

(Created page with "==Problem== (BorealBear) Find the last two digits of <math> 7^{81}-3^{81} </math>. ==Solution== This is a direct application of Euler's Totient Theorem. Since <math> \phi(10...")
 
m
Line 3: Line 3:
  
 
==Solution==
 
==Solution==
This is a direct application of Euler's Totient Theorem. Since <math> \phi(100)=40 </math>, this reduces to <math> 7^1-3^1\equiv \boxed{04}\pmod{100} </math>.
+
This is a direct application of Euler's Totient Theorem. Since <math> \phi(100)=40 </math>, this reduces to <math> 7^1-3^1\equiv \boxed{04}\pmod{100} </math>. -BorealBear

Revision as of 20:17, 23 April 2021

Problem

(BorealBear) Find the last two digits of $7^{81}-3^{81}$.

Solution

This is a direct application of Euler's Totient Theorem. Since $\phi(100)=40$, this reduces to $7^1-3^1\equiv \boxed{04}\pmod{100}$. -BorealBear